LSAT and Law School Admissions Forum

Get expert LSAT preparation and law school admissions advice from PowerScore Test Preparation.

 SLF
  • Posts: 40
  • Joined: Oct 01, 2013
|
#12927
Thank you. That is what I needed.
 srcline@noctrl.edu
  • Posts: 243
  • Joined: Oct 16, 2015
|
#22035
Hello,

I know there is already a thread on this, but I still couldn't understand the explanations. For me, the set up and diagramming the rules are fine, but its the inferences that are tripping me up.

First,in the C/N setup I dont understand how N is 7th. Is it N b/c of the 5th rule?
Second for the Suspect stack: my inferences were that S X V could not be first because W>S, Z>XY, (3rd & 4rth rules) also that suspect 7 is not W or Z (3/4 rules) Which means that 7 is S .

Also for question 8, I know I'm doing something wrong b/c B and C are both working in my setup. This game is really confusing me.

Thankyou
Sarah
 Jon Denning
PowerScore Staff
  • PowerScore Staff
  • Posts: 904
  • Joined: Apr 11, 2011
|
#22051
Hey Sarah,

Thanks for the question! This game is a little tricky at first, but as you probably noted in the other thread (this one, I presume: lsat/viewtopic.php?f=6&t=4022), a few key inferences really crush it.

You're correct about S in 7, and since S comes after W then we know from the rules that S did not confess. That means that spot 7 is a No Confess spot (whether you show that as N or as C or something else). So that's where that inference comes from.

Your Not Laws are also correct, so well done there!

Lastly, you ask about question 8...I thought at first perhaps you meant question 18 (since there's no question 8 in this game, being game 3 and all), but question 18 is a part of the fourth game, and the answer to 18 is (E) and you're debating (B) and (C) sounds like, so now I'm confused. Can you double check which question it is? In fact, with the setup you've got and what is hopefully some clarifying information above, give all the questions for this game another shot on your own and see how you do. I suspect you'll be able to crack them with another attempt :)

Thanks, and let me know how it goes!
 tomthomse@gmail.com
  • Posts: 5
  • Joined: May 15, 2020
|
#75966
Question-- why is T not considered as one who confessed? Just as no one confessed after W was questioned, W is allowed to, and does, confess. Since exactly 2 confess after T is questioned, why can't T be one of those 2?

Thanks for any clarification.
User avatar
 KelseyWoods
PowerScore Staff
  • PowerScore Staff
  • Posts: 1079
  • Joined: Jun 26, 2013
|
#76041
Hi Tom!

The last rule states that exactly two suspects confessed after T was questioned. Based on the scenario, we know that "any suspect who confessed did so while being questioned." That means that if T confessed, it would be during his questioning, not after his questioning.

So we actually don't know whether or not T confessed and you are correct--he absolutely could have confessed. But regardless of whether T confessed or not, we do know that two suspects after T must have confessed. So even if T did confess, he would not count as one of those two, since they must have been questioned after T.

Hope this helps!

Best,
Kelsey

Get the most out of your LSAT Prep Plus subscription.

Analyze and track your performance with our Testing and Analytics Package.